Question content area top Part 1 Three friends all live on the same street that runs west to east. lives blocks from . lives from . If the street is represented by a number line and ​'s house is located at​ 0, what are the possible locations for ​'s ​house?

Answers

Answer 1

Based on the information, it can be inferred that Carl's house is 6 blocks from Ann's house.

How to identify where each friend's house is?

To identify where each friend's house is, we must graph the information provided as shown below.

Once we have made the graph we will identify that Carl's house is 6 blocks east of Ann's house.

Note: This question is incomplete because there is some information missing. Here is the missing information:

Ben lives 4 blocks from Ann.Carl lives 2 blocks from Ben.Ann​'s house is located at​ 0.

Learn more about location in: https://brainly.com/question/11718756

#SPJ1

Question Content Area Top Part 1 Three Friends All Live On The Same Street That Runs West To East. Lives

Related Questions

solve this trigonometry question, please by an easy method.​

Answers

tan39=6/xx=6/tan39x=7.4

And

sin39=6/yy=6/sin39y=9.5

Answer:

x = 7.41 (nearest hundredth)

y = 9.53 (nearest hundredth)

Step-by-step explanation:

Trigonometric ratios

[tex]\sf \sin(\theta)=\dfrac{O}{H}\quad\cos(\theta)=\dfrac{A}{H}\quad\tan(\theta)=\dfrac{O}{A}[/tex]

where:

[tex]\theta[/tex] is the angleO is the side opposite the angleA is the side adjacent the angleH is the hypotenuse (the side opposite the right angle)

From inspection of the triangle:

[tex]\theta[/tex] = 39°O = 6A = xH = y

To find x, use the tan trigonometric ratio:

[tex]\implies \tan 39^{\circ}=\dfrac{6}{x}[/tex]

[tex]\implies x=\dfrac{6}{\tan 39^{\circ}}[/tex]

[tex]\implies x=7.409382939...[/tex]

Therefore, x = 7.41 (nearest hundredth)

To find y, use the sin trigonometric ratio:

[tex]\implies \sin 39^{\circ}=\dfrac{6}{y}[/tex]

[tex]\implies y=\dfrac{6}{\sin39^{\circ}}[/tex]

[tex]\implies y=9.534094374...[/tex]

Therefore, y = 9.53 (nearest hundredth)

Learn more about trigonometric ratios here:

https://brainly.com/question/27938163

https://brainly.com/question/27938731

AC is a tangent to the circle below at point B.
Calculate the sizes of angle x, angle y and angle z.
Justify each of your answers.​

Answers

[tex]y=78^{\circ}[/tex] (alternate interior angles theorem)

[tex]x=78^{\circ}[/tex] (alternate segment theorem)

[tex]z=24^{\circ}[/tex] (angles in a triangle add to 180 degrees)

PLEASE HELP ITS URGENT i need number 20 & 21
!!!!!

Answers

Answer:

Put the equations in the form y=mx+c and after that, take any two values for x, fill in the equation of y=mx+c to get the corresponding y coordinate and take any value for y, to get the corresponding x coordinate. For this example i took x=0 and y=0, feel free to try it again with any other value. If you get a fraction and don't what to work with fractions, just do a trial and error till you get a whole number (as i did for Question 21 for the x and y coordinates)

the inverse of the relation y=2x+3 can be obtained graphically by:

A. a reflection in the yaxis
B. a reflection in the line y= 1/2x
C. a reflection in the line y=√x
D. a reflection in the line y= x

Answers

Answer:

a reflection in the line y=x

Step-by-step explanation:

The inverse relation of an equation is when the x and y are swapped. For this reason it's going to be reflected over the y=x line. You can also use an online graphing tool to show this as well:

Help mee plsssssss hurry

Answers

Answer:

8

Step-by-step explanation:

Midpoint means half point of a certain shape. This means that if all the points of OPQ are midopoints of the bigger triangle, then OPQ has to be half the size of the big triangle. There are two ways to figure this out. The first way is to add all the numbers up to find the perimeter of the big triangle (6+6+4) which is 16. Then, you can do 16/2 to get the perimeter of the smaller triangle which is 8. The other method is to take each value of the bigger triangle and divide them by 2 (6/2, 6/2, 4/2). When you do that, you get (3, 3, 2). Now, you just add those values (3+3+2) which gets you 8 gor the smaller triangle. I hope this helps!

If 55% of the people at a certain conference are doctors, 49% are women, and 29% are female doctors, what is the probability that a person selected at random at this conference is a doctor or woman (or both)?

Answers

[tex]doctors = 55\% \\ woman = 49\% \\ both = 29\%[/tex]

[tex]p(a) =0.55 + 0.49 - 0.29 \\ p(a) = 0.75[/tex]

[tex]\frac{x^{2}-5x+6}{2x^{2}-7x+6 }[/tex]
whats the simplest form and the exclusions aka holes

Answers

Answer:

[tex]\frac{x-3}{2x-3}[/tex]. hole or removable discontinuity at x=2

Step-by-step explanation:

Well generally if you want the simplest form, you factor each the denominator and numerator and then see if you can cancel any of the factors out (because they're in the denominator and numerator)

So let's start by factoring the first equation:

[tex]x^2-5x+6[/tex]

Now let's find what ac is (it's just c since a=1...)

[tex]AC= 6[/tex]

List factors of -6

[tex]\pm1, \pm2, \pm3, \pm6[/tex].

Now we have to look for two numbers that add up to -5. It's a bit obvious here since there isn't many factors, but it's -2 and -3, and they're both negative since 6 is positive, and -5 is negative...

So using these two factors we get

[tex](x-2)(x-3)[/tex]

Ok now let's factor the second equation:

[tex]2x^2-7x+6[/tex]

Multiply a and c

[tex]AC = 12[/tex]

List factors of 12:

[tex]\pm1, \pm2, \pm3, \pm4, \pm6, \pm12[/tex].

Factors that add up to -7 and multiply to 12:

[tex]-3\ and\ -4[/tex]

Rewrite equation:

[tex]2x^2-4x-3x+6[/tex]

Group terms:

[tex](2x^2-4x)+(-3x+6)[/tex]

Factor out GCF:

[tex]2x(x-2)-3(x-2)[/tex]

Rewrite:

[tex](2x-3)(x-2)[/tex]

Now let's write out the equation using these factors:

[tex]\frac{(x-2)(x-3)}{(2x-3)(x-2)}[/tex].

Here we can factor out the x-2 and the simplified form is:

[tex]\frac{x-3}{2x-3}[/tex]

So we can "technically" define f(2) using the most simplified form, but it's removable discontinuity, so it has a hole as x=2. since it makes (x-2) equal to 0 (2-2) = 0.

Evaluate the following expression. -8 \times (-10 + (-7))−8×(−10+(−7))minus, 8, times, left parenthesis, minus, 10, plus, left parenthesis, minus, 7, right parenthesis, right parenthesis

Answers

The value of the given expression is 136

Simplifying an expression

From the question, we are to evaluate the give expression

The given expression is

−8×(−10+(−7))

The expression can be simplified as follows

−8×(−10+(−7))

−8×(−10−7)

−8×(−17)

= 136

Hence, the value of the given expression is 136

Learn more on Simplifying an expression here: https://brainly.com/question/723406

#SPJ1

What is the length of the hypotenuse ?

Answers

Answer:

3√2

Step-by-step explanation:

In a 45-45-90 triangle, the two legs are the same length while the hypotenuse is √2 times larger.

3*√2 = 3√2

31. For the following exercises, given each set of information, find a linear equation satisfying the conditions, if possible.
31. f(-1) = 4 and f(5) = 1

Answers

Answer:

The required linear equation satisfying the given conditions f(-1)=4 and f(5)=1 is [tex]$y=\frac{-1}{2} x+\frac{7}{2}$[/tex]

Step-by-step explanation:

It is given that f(-1)=4 and f(5)=1.

It is required to find out a linear equation satisfying the conditions f(-1)=4

and f(5)=1. linear equation of the line in the form

[tex]$$\left(y-y_{2}\right)=m\left(x-x_{2}\right)$$[/tex]

Step 1 of 4

Observe, f(-1)=4 gives the point (-1,4)

And f(5)=1 gives the point (5,1).

This means that the function f(x) satisfies the points (-1,4) and (5,1).

Step 2 of 4

Now find out the slope of a line passing through the points (-1,4) and (5,1),

[tex]$$\begin{aligned}&m=\frac{y_{2}-y_{1}}{x_{2}-x_{1}} \\&m=\frac{1-4}{5-(-1)} \\&m=\frac{-3}{5+1} \\&m=\frac{-3}{6} \\&m=\frac{-1}{2}\end{aligned}$$[/tex]

Step 3 of 4

Now use the slope [tex]$m=\frac{-1}{2}$[/tex] and use one of the two given points and write the equation in point-slope form:

[tex]$(y-1)=\frac{-1}{2}(x-5)$[/tex]

Distribute [tex]$\frac{-1}{2}$[/tex],

[tex]$y-1=\frac{-1}{2} x+\frac{5}{2}$[/tex]

Step 4 of 4

This linear function can be written in the slope-intercept form by adding 1 on both sides,

[tex]$$\begin{aligned}&y-1+1=\frac{-1}{2} x+\frac{5}{2}+1 \\&y=\frac{-1}{2} x+\frac{5}{2}+\frac{2}{2} \\&y=\frac{-1}{2} x+\frac{7}{2}\end{aligned}$$[/tex]

So, this is the required linear equation.

In a continuous series, mean(x) = 80, assumed mean(A) = 60 and
EF=40 then find the value of EFd.​

Answers

Using the given information, the value of Σfd is 800

Calculating mean using Assumed mean

From the question, we are to determine the value of Σfd

The formula for mean, using the assumed mean method is given by

[tex]\bar x = A + \frac{\sum fd}{\sum f}[/tex]

Where [tex]\bar x[/tex] is the mean

A is the assumed mean

From the given information,

[tex]\bar x = 80[/tex]

[tex]A = 60[/tex]

[tex]\sum f = 40[/tex]

Putting the parameters into the equation, we get

[tex]80 = 60 + \frac{\sum fd}{40}[/tex]

[tex]80-60 = \frac{\sum fd}{40}[/tex]

[tex]20 = \frac{\sum fd}{40}[/tex]

[tex]\sum fd = 20 \times 40[/tex]

[tex]\sum fd = 800[/tex]

Hence, the value of Σfd is 800

Learn more on Mean here: https://brainly.com/question/20118982

#SPJ1

Find the UY segment measurement if

bold increment bold italic X bold italic Y bold italic Z bold approximately equal to bold increment bold italic U bold italic Y bold italic W
.
26

20

24

10

Answers

Triangle XYZ = UYW

We know that the 2 triangles are similar using angles Theorem

Let us find XZ to determine UY (for confirmation):

To determine XZ, use the Pythagorean Theorem:

[tex]a^{2} + b^{2} = c^{2} \\ OR\\leg^{2} +leg^{2} = Hypotenuse^{2} \\24^{2} + b^{2} = 26^{2} \\576+b^{2} =676\\b^{2} =100\\b=10[/tex]

You can see that XZ = WU

Now find UY= 26

Corresponding angles

Hope it helps!

HELP HELP HELP

Just 5 quick algebra 1 questions for 100 points!

Answers

1. x = a

2. y = b

3. The slope of a vertical line is undefined..

4. Change in y/change in x (m) or rise/run

5. Rewrite the equation in slope-intercept form to find the value of the y-intercept. The value of x will always be 0.

What is the Equation of a Line?

The equation, y = mx + b, in slope-intercept form, defines a line, where m is the slope and the y-intercept = b.

1. Slopes of vertical lines are always undefined, therefore, for a vertical line that contains (a, b), the equation would be: x = a.

2. Horizontal lines have a slope of 0, therefore, the equation of a horizontal line that contains the point, (a, b), will  be: y = b.

3. The slope of a vertical line is undefined.

4. Two ways of finding the slope if we are given two points on a line are:

Change in y/change in x (m) = y2 - y1 / x2 - x1

or

rise/run

5. If we have an equation in point-slope form, for example, y - 2 = 2(x - 4), rewrite the equation in slope-intercept form and find the value of the y-intercept:

y - 2 = 2x - 8

y = 2x - 8 + 2

y = 2x - 6

The y-intercept = -6, therefore, the coordinates of the y-intercept would be: (0, -6).

Learn more about the equation of a line on:

https://brainly.com/question/19417700

#SPJ1

(1) Triangle ABC is an isosceles triangle.
Find the altitude h.
B
45°

Answers

Step-by-step explanation:

45+45

90

I guess is the answer

the interest paid on a $8,000 loan is $600, a rate of 4%
how many years did it take to pay back the loan? round to 1 decimal.

Answers

It took 1.9 years to pay back the loan.

What is Interest ?

Interest is the amount of money given or received when a certain sum of amount is received as a loan or given or deposited for investment.

It is given that

Principal amount of loan = $ 8000

Interest paid = $ 600

Rate = 4%

Time Period = ?

Assuming Simple Interest has been applied

I = ( P* R* T) /100

600 = ( 8000 * 4 * T ) / 100

60000 = 8000 * 4 * T

T = 1.875 years

T = 1.9 years rounded to 1 decimal

T = 22.5 months

Therefore it took 1.9 years to pay back the loan.

To know more about Interest

https://brainly.com/question/13324776

#SPJ1

select the true statement about triangle abc

Answers

Answer:

Step-by-step explanation:

Please post a picture of the triangle and the choices.

Imagine that the figure shown is moved 4 units to the left and 3 units down.


Which point is not a vertex of the transferred image?

(-2, -3)

(-4, -3)

(-3, -1)

(-2, 1)

Answers

Answer:

(-4,-3)

Step-by-step explanation:

Just check each vertex point and move them by instruction, write them down and do elimination and you'll find the answer

PLEASE HELP!!! FINDING PROBABILITY MATH!!! expert help would be nice
i need the answer to question 2, 3, and 4. question 1's answer is 9/28 & 32.1%.

Answers

The probability that a person wins the game is 32.1%

How to illustrate the probability?

Based on the information given, the following can be depicted. It should be noted that there are 6 sides as well as 4 cards.

Therefore, the numbers on the dice i.e from 1 - 6 will be represented 4 times each. This gives a total of (4 × 6) = 24. There are also 4 cards. The total in sample space will now be:

= 24 + 4 = 28

The frequency table will be such that 28 or more have a relative frequency of 9. Therefore, the probability that a person wins the game will be:

= 9/28 = 32.1%

When you win 25% of the time, this illustrates that the number of products picked will be:

= 25% × 28

= 7 products.

The probability of participants achieving a winning score of 36 or higher in four consecutive attempts will be:

= 1/6⁴ = 1/1296

Learn more about probability on:

https://brainly.com/question/24756209

#SPJ1

there were three parts to Rita's race. she ran the first part, which was 4/9 of the total distance, in 20 minutes. She ran the second part, which was 2/5 of the remaining distance, in 12 minutes. She finally ran the third part in 15 minutes at a speed of 300 meters per minute.
a) How long was the third part of the race?

Answers

20+12+15= 32+15 = 47 minutes in total

300 meters per minute

300 * 47 = 14100 meters in total

with this we can see the first race was 4/9 that distance meaning the first race was 6266.6 meters long

14100 - 6266.6 = 7833.3

2/5 of 7833.3 is 3133.32

therefore 7833.3 - 3133.32 will give the distance of the third part of the race

meaning of course the answer is 4699.98 or rounding up 4700 meters for the third race

Plis help! Will give brainliest!

Answers

Answer: 7) Step 1 8) Step

Step-by-step explanation:

The answer to 7 is 11. The error starts in step 1, as you can see youre supposed to do 8 divided by 2 plus 3 times 3 minus 2, instead it seems like 3+3 were added instead of becoming 9 thus the answer becomes 7.

The answer to 8 is also 11. 18-14/2 is 11.

A motorcycle consumes 5 gallons of gas for 300 miles and 7 gallons for 420 miles. Calculate the rate of change per gallon of gas.

Answers

Thus, the rate of change per gallon of gas is 60 miles per gallon.

A motorcycle consumes 5 gallons of gas for 300 miles and 7 gallons for 420 miles.

What is rate of change?

Rate of change is defined as the change in value with rest to the time is called rate of change.

Now,

Rate of change = Difference B/w volume (i.e. in gallons)  /difference in miles
= 420-300/7-2
= 60 gallons/mile

Thus, The required rate of change per gallon of gas is 60 miles per gallon.

Learn more about rate of change here: https://brainly.com/question/13103052

#SPJ1

Select the function that represents a geometric sequence.
OA. A(n) = P+ (n-1)i P, where n is a positive integer
B. A(n) = (n-1)(P. )", where n is any real number
O C. A(n) = P(1 + i)-1, where n is a positive integer
OD. A(n) = n+ (P-1)i P, where n is a positive integer

Answers

The function that represents a geometric sequence is given by:

C. [tex]A(n) = P(1 + i)^{n-1}[/tex], where n is a positive integer.

What is a geometric sequence?

A geometric sequence is a sequence in which the result of the division of consecutive terms is always the same, called common ratio q.

The nth term of a geometric sequence is given by:

[tex]a_n = a_1q^{n-1}[/tex]

In which [tex]a_1[/tex] is the first term.

Following this pattern, a function that is also a geometric sequence is:

C. [tex]A(n) = P(1 + i)^{n-1}[/tex], where n is a positive integer.

For the function, we have that:

[tex]a_1 = P, q = 1 + i[/tex].

More can be learned about geometric sequences at https://brainly.com/question/11847927

#SPJ1

Consider the paragraph proof. given: d is the midpoint of ab, and e is the midpoint of ac. prove:de = one-halfbc on a coordinate plane, triangle a b c is cut by line segment d e. point d is the midpoint of side a b and point e is the midpoint of side a c. point a is at (2 b, 2 c), point e is at (a b, c), point c is at (2 a, 0), point b is at (0, 0), and point d is at (b, c). it is given that d is the midpoint of ab and e is the midpoint of ac. to prove that de is half the length of bc, the distance formula, d = startroot (x 2 minus x 1) squared (y 2 minus y 1) squared endroot, can be used to determine the lengths of the two segments. the length of bc can be determined with the equation bc = startroot (2 a minus 0) squared (0 minus 0) squared endroot, which simplifies to 2a. the length of de can be determined with the equation de = startroot (a b minus b) squared (c minus c) squared endroot, which simplifies to ________. therefore, bc is twice de, and de is half bc. which is the missing information in the proof? a 4a a2 4a2

Answers

The missing information in the proof is a which is option A.

Given d is the mid point of ab and e is the mid point of ac. Coordinates are point a (2b,2c), point e (ab, c), point c (2a, 0),point b (0,0), point d (b, c).

We have to find the missing proof in the solution.

To find the missing figure we have to just find the distance between point d and point e.

Distance formula for determining the distance between two points on a coordinate plane is given as :

d=[tex]\sqrt{(y_{2} -y_{1} )^{2} +(x_{2} -x_{1} )^{2} }[/tex]

where ([tex]x_{1} ,y_{1} ) (x_{2} ,y_{2} )[/tex] are the coordinates at the end of line.

DE=[tex]\sqrt{(a+b-b)^{2} +(c-c)^{2} }[/tex]

Simplifying this we get:

DE=[tex]\sqrt{(a^{2} +0^{2} }[/tex]

=[tex]\sqrt{a^{2} }[/tex]

=a

Hence the missing proof is a which is the distance or length of DE..

Learn more about distance at https://brainly.com/question/2854969

#SPJ4

Evan has a storage unit that he keeps all his garden tools in. It measures 4 feet by 312 feet by 2 feet.A prism has a length of 4 feet, height of 2 feet, and width of 3 and one-half feet.What is the maximum amount of supplies that the storage unit can hold

Answers

The maximum amount of supplies that the storage unit can hold is 28 ft³

How to calculate the volume of a rectangular prism?

For us to calculate the volume or amount of space in Evan's prism, we will first of all calculate the volume of rectangular prisms. Formula is;

V = Length × width × height.

We are given;

length = 4 feet

width =  3.5 feet

height of the prism = 2 feet.

Thus;

V = 4 * 3.5 * 2

V = 28 ft³

Thus, the maximum amount of supplies that the storage unit can hold is 28 ft³

Read more about Volume of Rectangular Prism at; https://brainly.com/question/4062480

#SPJ1

What is the sum of a° + b°?

180°

We need more information to solve this problem.

The answer depends on the values of the individual angles.

360°

Answers

Answer:

a=70°

b=120°

we know,

a+b

=70+120

=190

April works as an emergency medical technician (EMT) and is a photographer. As an EMT she earns $19 per hour. Last week she worked t hours as an EMT and her friend paid her $100 for a photoshoot. Write an expression to represent the total amount April earned last week.

Answers

The expression that represents the total amount earned last week $100 + $19t.

What is the expression?

The expression is a function of the amount her friend paid for the photoshoot, the hours worked and the wages per hour.

Total pay = (amount earned per hour x total hours worked) + amount paid for photoshoot

($19 x t) + $100

$19t + $100

To learn more about mathematical expressions, please check: https://brainly.com/question/11980704

#SPJ1

HELP ASAP
What is the solution to the inequality below?
12+x23(x-6)
OA. x≤ 12
OB. x≤ 5
OC. x≤ 15
OD. x≤9

Answers

The solution to the inequality is x ≤ 15

How to solve the inequality?

The inequality is given as:

12 + x ≥ 3(x-6)

Open the bracket

12 + x ≥ 3x - 18

Evaluate the like terms

-2x ≥ -30

Divide by -2

x ≤ 15

Hence, the solution to the inequality is x ≤ 15

Read more about inequality at:

https://brainly.com/question/24372553

#SPJ1

f(x)=3x-2
h(x)=3x
g(x)=x squared
find the value of x when f(x)=19

Answers

The value of f(19) in the given equation is 55.

What is the function of a linear equation?

The function of a linear equation illustrates a straight line graph and for a given set of input into the function, there is usually a specific output.

Given that:

f(x) = 3x - 2

We are to find the f(x) = 19.

So;

f(19) = 3(19) - 2

f(19) = 57 - 2

f(19) = 55

Learn more about the function of  a linear equation here:

https://brainly.com/question/14323743

#SPJ1

Pls help to find the coordinate on the line. Will mark brainliest. Thank you!!!

Answers

The coordinate of point R is 11/40, so the correct option is C.

How to find the coordinate of point R?

On the number line we can see that:

M = -1/4

T = 5/8

There are 5 segments between T and M, and the difference between T and M is:

5/8 - (-1/4) = 5/8 + 2/8 = 7/8

The measure of each segment will be equal to:

m = (7/8)/5 = 7/(8*5) = 7/40

Now, coordinate R is 2 segments to the left of T, so the coordinate of point R is given by:

R = 5/8 - 2*(7/40)

R = 25/40 - 14/40 = 11/40

The correct option is C.

If you want to learn more about number lines:

https://brainly.com/question/4727909

#SPJ1

Multiply Conjugates Using the Product of Conjugates Pattern
In the following exercises, multiply each pair of conjugates using the Product of Conjugates Pattern
329. (9c + 5)(9c − 5)

Answers

Answer:

The product is the difference of squares is [tex]$$\left(9c+5\right)\left(9c-5\right)=81{{c}^2}-25$$[/tex]

Step-by-step explanation:

Explanation

The given expression is (9 c+5)(9 c-5).We have to multiply the given expression.Square the first term 9c. Square the last term 5 .

[tex]$$\begin{aligned}&(9 c+5)(9 c-5)=(9 c)^{2}-(5)^{2} \\&(9 c+5)(9 c-5)=81 c^{2}-25\end{aligned}$$[/tex]

Other Questions
A solution is made by dissolving 3.14 g of NaCl in 79.9 mL of water. What is the concentration of sodium chloride in units of weight/volume percent? Group of answer choices 3.78% (w/v) NaCl 96.1% (w/v) NaCl 3.93% (w/v) NaCl 0.0393% (w/v) NaCl The following diagram shows the branching tree for four kingdoms and some of their shared derived characteristics. (Common Cell) Protists Plants Fungi Animals What shared characteristic can be written at point X? Use complete sentences to explain your answer. Describe the composition and the main parts of a comet. Scientists want to test a new pair of running shoes. A speed test is performed with two separate groups of participants. The treatment group will wear the new pair of running shoes, while the control group will not. It is believed that age and height may affect speed. Which of the following would be most effective in controlling the confounding variables, such as age and height, in this study? The sum of the measures of the angles of a triangle is 180. The sum of the measures of the second and third angles is fourtimes the measure of the first angle. The third angle is 12 more than the second. Let x, y, and z represent the measures ofthe first, second, and third angles, respectively. Find the measures of the three angles. x, 3x and 5x ,find the sum Which answer shows y+2x is asthma is cased by virus 2.what regulatory standards dictate how you would handle requests for substitutions in long-term care, and what do they say Find the gradients of lines A and B.B14112Hol8NO 9 go6412-3-2-10-2-4-6A2 3 4 57x what is the purpose of genetic screening The water is flowing through the horizontal constricted pipe. The pressure at one end is 4500Pa, speed is 3m/s and area of cross-section is A, Calculate the speed and pressure at another end where the area of cross-section is A/3. A toy dart gun uses a spring with a spring constant of 35 N/m. To use the dart gun, you compress the spring by pushing in a dart of mass 0.002 kg. a. If you compress the spring by 0.02 m, what force is the spring exerting on the dart? b. With the spring compressed 0.02 m, how much elastic potential energy is stored in the spring? c. If you release the spring, it pushes the dart forward. What is the kinetic energy of the dart when it reaches the natural length of the spring? Explain how you found the answer. d. What is the speed of the dart when it reaches the natural length of the spring? which of the following needs to be cited within the text of a paper? select all that apply A company emphasizes ________ in its strategic planning where it develops four alternative futures based on two key variables: the cost of energy and the extent of growth opportunities in emerging markets. In skunks, stripes (S) is dominant to spots (s) and black coat color (B) is dominant to brown coat color (b). If a skunk that is homozygous for stripes and heterozygous for black coat color is crossed to a skunk that is homozygous for spots and heterozygous for black coat color, what genotypic and phenotypic ratios would you expect to observe among the offspring Write Five metaphors about something you do outside of school.(make it up) What is the structure of Susan B. Anthonys On Womens Right to Vote?A. Compare and contrastB. ArgumentativeC. Cause and effectD. Problem and Solution Which is an example of enjambment?A. The waves move their long row of scythes over the long beach.B. It happens as we watch.C. It happens as we sleep,the way the clock's handsD. none of the above Express each ratio as a unit rate. Round to the nearest tenth, if necessary.140 miles on 6 gallonsa.23.3 mi/galc.0.042 mi/galb.840 mi/gald.13.6 mi/galPlease select the best answer from the choices providedABCD